which of the following are possible values of r?
[tex] {r}^{2 } = \frac{3}{16} [/tex]

Answers

Answer 1

Answer:

[tex]r=\frac{\sqrt{3} }{4}[/tex]    and    [tex]r=-\frac{\sqrt{3} }{4}[/tex]

Step-by-step explanation:

when you solve for r in the given equation, you need to apply the square root property, which gives positive and negative answers (both should therefore be considered):

[tex]r^2=\frac{3}{16} \\r=+/-\sqrt{\frac{3}{16}} \\r=+/-\frac{\sqrt{3} }{4}[/tex]

then you need to include these two possible solutions:

[tex]r=\frac{\sqrt{3} }{4}[/tex]    and    [tex]r=-\frac{\sqrt{3} }{4}[/tex]


Related Questions

George buys a pizza he eats 3-8 of pizza for lunch and 1-4 of pizza for dinner what fraction of pizza has George eaten

Answers

Answer:

George has eaten 5/8 of the pizza

Step-by-step explanation:

Step 1: Multiple 1/4 by 2 so it shares a common denominator with 3/8

1.4 x 2 = 2/8

Step 2: Because they share a denominator you can add the numerator together

2/8 + 3/8 = 5/8

Therefore George has eaten 5/8(Five Eigths) of the pizza

George has eaten 5 by 8 of the pizza

The calculation is as follows:

Here we have to Multiple 1 by 4 with 2 so it shares a common denominator with 3 by 8

[tex]1.4 \times 2 = 2\div 8[/tex]

Now  

since they share a denominator you can add the numerator together

So,  [tex]\frac{2}{8} + \frac{3}{8} = \frac{5}{8}[/tex]

Learn more: https://brainly.com/question/17429689?referrer=searchResults

Need Assitance
*Show Work*​

Answers

Answer:

66 2/3 %

Step-by-step explanation:

First find the students not in the 8th grade

24 - 8 = 16

16 students are not in the 8th grade

Take the fraction of the students not in the 8th grade over the total

16/24 = 2/3

Change to a decimal

.66666666666

Multiply by 100 to change to a percent

66.666666%

66 2/3 %

Answer:

66.67% of students are not in eighth grade

Step-by-step explanation:

8/24=1/3

1/3=0.33333333333

1-0.33333333333=0.66666666667

0.66666666667=66.67%

What is the equation of the line in the following graph?

Answers

Answer:

2 . y=-1

Step-by-step explanation:

m=0  (it is a straight line)

use (-6,-1) in y=mx+b

-1=0(-6)+b

-1=b

equation is now

y=0(x)-1

y=-1

If 75% of a number 60, what is the number?

Answers

Answer:

80

Step-by-step explanation:

75                60

------     =      ------

100                x

100 x 60 = 6000

75x = 6000

x =  80

Given a sample of 35, what is the sample standard deviation of a pair of jeans if the 90% confidence interval is [37.14, 42.86]

Answers

Answer:

10.295

Step-by-step explanation:

Using the value for calculating the confidence interval as given;

CI = xbar + Z*σ/√n

xbar  is the mean = 37.14+42.86/2

xbar= 80/2

xbar = 40

Z is the z-score at the 90% confidence = 1.645

σ is the standard deviation

n is the sample size = 35

Given the confidence interval CI as [37.14, 42.86]

Using  the maximum value of the confidence interval to get the value of the standard deviation, we will have;

42.86 =  xbar + Z*σ/√n

42.86 = 40 + 1.645* σ/√35

42.86-40 = 1.645*σ/√35

2.86 = 1.645*σ/√35

2.86/1.645 = σ/√35

1.739 = σ/√35

1.739 = σ/5.92

σ= 1.739*5.92

σ = 10.295

Hence, the sample standard deviation of a pair of jeans is 10.295

If f(x)=2x-6and g(x)=3x+9 find (f+g)(x)

Answers

Answer:

(f+g)(x) = 5x + 3

Step-by-step explanation:

(f+g)(x) is the sum (term by term) of f(x) and g(x):

(f+g)(x) = 2x - 6 + 3x + 9

Combining like terms, we get

(f+g)(x) = 5x + 3

Answer:

(f+g)(x)= 5x+3

Step-by-step explanation:

The question asks us to find (f+g)(x). In other words, the sum of f(x) and g(x).

f(x) + g(x)

We know that f(x)= 2x-6 and g(x)=3x+9. Therefore, we can substitute the expressions in.

(2x-6) + (3x+9)

Now, simplify by combining like terms. Add the terms with variables, then the terms without variables.

(2x+3x) + (-6+9)

Add 2x and 3x.

5x + (-6 + 9)

Add -6 and 9.

5x + 3

If f(x)=2x-6and g(x)=3x+9, then (f+g)(x) is 5x+3

What is the factored form of the binomial expansion x3 + 9x2 + 27x + 27?
(x + 3)3
(x - 3)3
(x + 9)3
(X - 9)3

Answers

Answer:

A

Step-by-step explanation:

the factored form of the binomial expansion x^3 + 9x^2 + 27x + 27 is (x+3)^3

Find usubscript10 in the sequence -23, -18, -13, -8, -3, ...

Answers

Step-by-step explanation:

utilise the formula a+(n-1)d

a is the first number while d is common difference

Answer:

22

Step-by-step explanation:

Using the formular, Un = a + (n - 1)d

Where n = 10; a = -23; d = 5

U10 = -23 + (9)* 5

U10 = -23 + 45 = 22

I need help please, show work

Answers

Answer:

24 and 32 ft or 32 and 24 ft

Step-by-step explanation:

Perimeter of rectangle(p)=2(l+b)

or, 112/2=l+b

Therefore, l+b=56

Now,

diagonal(d)=40

By pythogoras theorem,

h^2=p^2+b^2 (d=h here)

40^2=l^2+b^2

Now,

Square l+b=56

(l+b)^2=56^2

l^2+2lb+b^2=3136

2lb=3136-1600

lb=1536/2

Therefore, lb=768

b=768/l

Now,

Perimeter of rectangle(p)=2(l+b)

l+b=56

l+768/l=56

l^2+768=56l

l^2+768-56l=0

Factoring,

(l - 32) (l - 24) = 0

Either l= 32 or l = 24

When l=32,

l+b=56

32+b=56

b=24

When l=24

l+b=56

24+b=56

b=32

So the dimensions of the dance floor are 24 and 32 ft or 32 and 24 ft.

Answer:

24 ft x 32 ft

Step-by-step explanation:

[tex]2x+2y=112[/tex]

[tex]\sqrt{x^{2}+y^{2} } =40[/tex]

Graph the equations

Find the point where they intersect

Answer is 24 ft and 32 ft

If 3, 9, 10, x are in proportion find the value of x.​

Answers

Answer:

[tex]x = 30[/tex]

Step-by-step explanation:

If 3, 9, 10, x are in proportion. Then,

[tex]x \times 3 = 10 \times 9[/tex]

[tex]x \times 3 = 90[/tex]

[tex]x = \frac{90}{3} [/tex]

[tex]x = 30[/tex]

Hope it is helpful.....

Step-by-step explanation:

Given that 3, 9, 10, x are in proportion <br> product of exterms = Product of means <br>

<br>

3x=9×10

X=(9x10)/3

X=90/3

X=30

If the solutions for a quadratic equation are -2 and 5 what is the equation

Answers

Answer:

f(x) = x^2 - 3x -10

Step-by-step explanation:

If the solutions are {-2, 5}, the factors of the quadratic are (x + 2) and (x - 5).

The equation is f(x) = (x + 2)(x - 5) = x^2 - 3x -10

Question 2: Jamie has a jar of coins containing the same number of nickels, dimes and quarters. The total value of the coins in the jar is 13.20. How many nickels does Jamie have?

Answers

Answer:

?

Step-by-step explanation:

Answer:

33

Step-by-step explanation:

Let "x" be the number of nickels, of dimes, and of quarters.

The value of the nickels is 5x cents.

The value of the dimes is 10x cents

The value of the quarters is 25x cents.

Equation:

Value of nickels + Value of dimes + Value of quarters =1320 cents

5x + 10x + 25x = 1320

Sove for "x". Then you will know the number of each coin.

What’s the next number is this series 31,29,24,22,17,

Answers

Answer:

15

Step-by-step explanation:

Just find out the pattern.  It is decreasing, so let's find out how much it is decreasing by.

31-29= -2

29-24= -5

24-22= -2

22-17= -5

The pattern is -2, -5, -2, -5... So the next one should be -2 again!  17-2=15.

Remember, a pattern doesn't always have to be subtracting, adding, dividing, or multiplying at a constant number!  It can switch between two, like this problem!

Write 8x8x88888 as power

Answers

Answer:

8[2]×88888

Step-by-step explanation:

[8×8]=8[2]×88888

Halla x si:

a) 4√5 b) √5 c) 4√3 d) 4 e) 4√2

Answers

Answer:

Option A. 4√5

Step-by-step explanation:

To obtain the value of x, we must first obtain the value of y as shown in the attached photo.

The value of y can be obtained by using the pythagoras theory as illustrated below:

In this case y is the longest side i.e the Hypothenus.

y² = 4² + [4√3]²

y² = 4² + [4² × (√3)²]

y² = 4² + [4² × 3]

y² = 16 + [16 × 3]

y² = 16 + 48

y² = 64

Take the square root of both side

y = √64

y = 8

Finally, we shall determine the value of x by using the pythagoras theory as illustrated below.

Note: x is the longest side i.e the Hypothenus in this case.

x² = 4² + 8²

x² = 16 + 64

x² = 80

Take the square root of both side

x = √80

x = √(16 × 5)

x = √16 × √5

x = 4√5

Therefore, the value of x is 4√5.

Factor the trinomial below. x^2 + 5x – 24 A. (x – 8)(x + 3) B. (x – 4)(x + 6) C. (x – 3)(x + 8) D. (x – 6)(x + 4)

Answers

Answer:

The answer is option C

Step-by-step explanation:

x² + 5x - 24

To factorize first write 5x as a difference so that when subtracted will give you 5 and when multiplied will give you - 24

That's

x² + 8x - 3x - 24

Factorize x out

That's

x( x + 8) - 3(x + 8)

Factor x + 8 out

We have the final answer as

(x + 8)(x - 3)

Hope this helps you

Answer:(x-3)(x+8)

Step-by-step explanation:

On Halloween, a man presents a child with a bowl containing eight different pieces of candy. He tells her that she may have three pieces. How many choices does she have

Answers

Answer:

[tex]56[/tex] choices

Step-by-step explanation:

We know that we'll have to solve this problem with a permutation or a combination, but which one do we use? The answer is a combination because the order in which the child picks the candy does not matter.

To further demonstrate this, imagine I have 4 pieces of candy labeled A, B, C, and D. I could choose A, then C, then B or I could choose C, then B, then A, but in the end, I still have the same pieces, regardless of what order I pick them in. I hope that helps to understand why this problem will be solved with a combination.

Anyways, back to the solving! Remember that the combination formula is

[tex]_nC_r=\frac{n!}{r!(n-r)!}[/tex], where n is the number of objects in the sample (the number of objects you choose from) and r is the number of objects that are to be chosen.

In this case, [tex]n=8[/tex] and [tex]r=3[/tex]. Substituting these values into the formula gives us:

[tex]_8C_3=\frac{8!}{3!5!}[/tex]

[tex]= \frac{8*7*6*5*4*3*2*1}{3*2*1*5*4*3*2*1}[/tex] (Expand the factorials)

[tex]=\frac{8*7*6}{3*2*1}[/tex] (Cancel out [tex]5*4*3*2*1[/tex])

[tex]=\frac{8*7*6}{6}[/tex] (Evaluate denominator)

[tex]=8*7[/tex] (Cancel out [tex]6[/tex])

[tex]=56[/tex]

Therefore, the child has [tex]\bf56[/tex] different ways to pick the candies. Hope this helps!

A model rocket is launched with an initial velocity of 240 ft/s. The height, h, in feet, of the rocket t seconds after the launch is given by
h = −16t2 + 240t.
How many seconds after launch will the rocket be 390 ft above the ground? Round to the nearest hundredth of a second.

s (smaller value)
s (larger value)

Answers

Answer:

About 1.85 seconds and 13.15 seconds.

Step-by-step explanation:

The height (in feet) of the rocket t seconds after launch is given by the equation:

[tex]h = -16t^2 + 240 t[/tex]

And we want to determine how many seconds after launch will be rocket be 390 feet above the ground.

Thus, let h = 390 and solve for t:

[tex]390 = -16t^2 +240t[/tex]

Isolate:

[tex]-16t^2 + 240 t - 390 = 0[/tex]

Simplify:

[tex]8t^2 - 120t + 195 = 0[/tex]

We can use the quadratic formula:

[tex]\displaystyle x = \frac{-b\pm\sqrt{b^2 -4ac}}{2a}[/tex]

In this case, a = 8, b = -120, and c = 195. Hence:

[tex]\displaystyle t = \frac{-(-120)\pm \sqrt{(-120)^2 - 4(8)(195)}}{2(8)}[/tex]

Evaluate:

[tex]\displaystyle t = \frac{120\pm\sqrt{8160}}{16}[/tex]

Simplify:

[tex]\displaystyle t = \frac{120\pm4\sqrt{510}}{16} = \frac{30\pm\sqrt{510}}{4}[/tex]

Thus, our two solutions are:

[tex]\displaystyle t = \frac{30+ \sqrt{510}}{4} \approx 13.15 \text{ or } t = \frac{30-\sqrt{510}}{4} \approx 1.85[/tex]

Hence, the rocket will be 390 feet above the ground after about 1.85 seconds and again after about 13.15 seconds.

What is the range of possible sizes for side z?
Pro
Pro
Tea
2
4.1
1.3
Stuck? Watch a video or use a hint.
Reportage

Answers

Answer:

2.8 < x < 5.4

Step-by-step explanation:

Given the triangle with two known sides, 4.1 and 1.3, the range of possible values of the third side, x, can be ascertained by considering the triangle inequality theorem.

According to the theorem, when you add any two of the angles in a triangle, it should give you a value greater than the third side.

If a, b, and c are 3 sides of a triangle, the theorem implies that:

a + b > c.

Therefore, a - b < c < a + b

We can use this logic to find the possibly values of x in the given triangle above.

Thus,

4.1 - 1.3 < x < 4.1 + 1.3

2.8 < x < 5.4

Range of possible sizes of x is 2.8 < x < 5.4

A television screen has a length to width ratio of 8 to 5 and a perimeter of 117 inches. What is the diagonal measure of the screen (to the nearest tenth of an inch)?

Answers

Answer:

[tex]D = 42.5\ inch[/tex]

Step-by-step explanation:

Given

[tex]L = Length[/tex] and [tex]W = Width[/tex]

[tex]L:W = 8: 5[/tex]

[tex]Perimeter = 117[/tex]

Required

Determine the Diagonal

First, the dimension of the screen has to be calculated;

Recall that; [tex]L:W = 8: 5[/tex]

Convert to division

[tex]\frac{L}{W} = \frac{8}{5}[/tex]

Multiply both sides by W

[tex]W * \frac{L}{W} = \frac{8}{5} * W[/tex]

[tex]L = \frac{8W}{5}[/tex]

The perimeter of a rectangle:

[tex]Perimeter = 2(L+W)[/tex]

Substitute [tex]L = \frac{8W}{5}[/tex]

[tex]Perimeter = 2(\frac{8W}{5}+W)[/tex]

Take LCM

[tex]Perimeter = 2(\frac{8W + 5W}{5})[/tex]

[tex]Perimeter = 2(\frac{13W}{5})[/tex]

Substitute 117 for Perimeter

[tex]117 = 2(\frac{13W}{5})[/tex]

[tex]117 = \frac{26W}{5}[/tex]

Multiply both sides by [tex]\frac{5}{26}[/tex]

[tex]\frac{5}{26} * 117 = \frac{26W}{5} * \frac{5}{26}[/tex]

[tex]\frac{5 * 117}{26} = W[/tex]

[tex]\frac{585}{26} = W[/tex]

[tex]22.5 = W[/tex]

[tex]W = 22.5[/tex]

Recall that

[tex]L = \frac{8W}{5}[/tex]

[tex]L = \frac{8 * 22.5}{5}[/tex]

[tex]L = \frac{180}{5}[/tex]

[tex]L = 36[/tex]

The diagonal of a rectangle is calculated using Pythagoras theorem as thus;

[tex]D = \sqrt{L^2 + W^2}[/tex]

Substitute values for L and W

[tex]D = \sqrt{36^2 + 22.5^2}[/tex]

[tex]D = \sqrt{1296 + 506.25}[/tex]

[tex]D = \sqrt{1802.25}[/tex]

[tex]D = \sqrt{1802.25}[/tex]

[tex]D = 42.4529150943[/tex]

[tex]D = 42.5\ inch[/tex] (Approximated)

Identifying the Property of Equality

Quick

Check

Identify the correct property of equality to solve each equation.

3+x= 27

X/6 = 5

Answers

Answer:

a) Compatibility of Equality with Addition, b) Compatibility of Equality with Multiplication

Step-by-step explanation:

a) This expression can be solved by using the Compatibility of Equality with Addition, that is:

1) [tex]3+x = 27[/tex] Given

2) [tex]x+3 = 27[/tex] Commutative property

3) [tex](x + 3)+(-3) = 27 +(-3)[/tex] Compatibility of Equality with Addition

4) [tex]x + [3+(-3)] = 27+(-3)[/tex] Associative property

5) [tex]x + 0 = 27-3[/tex] Existence of Additive Inverse/Definition of subtraction

6) [tex]x=24[/tex] Modulative property/Subtraction/Result.

b) This expression can be solved by using the Compatibility of Equality with Multiplication, that is:

1) [tex]\frac{x}{6} = 5[/tex] Given

2) [tex](6)^{-1}\cdot x = 5[/tex] Definition of division

3) [tex]6\cdot [(6)^{-1}\cdot x] = 5 \cdot 6[/tex] Compatibility of Equality with Multiplication

4) [tex][6\cdot (6)^{-1}]\cdot x = 30[/tex] Associative property

5) [tex]1\cdot x = 30[/tex] Existence of multiplicative inverse

6) [tex]x = 30[/tex] Modulative property/Result

Answer:

3 + x = 27

✔ subtraction property of equality with 3

x over 6  = 5

✔ multiplication property of equality with 6

15. The height and yolume of a cylinder are 4cm and 616cm respectively. Calculate the diameter of the base. (take t = 27 쪽 A. 7cm B. 154cm C. 14cm D. 64cm​

Answers

Step-by-step explanation:

Given that,

Height of cylinder = 4 cmVolume of cylinder = 616 cm³

To find,

Diameter of the base = ?

Firstly we'll find the base radius of the cylinder.

[tex]\longmapsto\rm{V_{(Cylinder)} = \pi r^2h}\\[/tex]

According to the question,

[tex]\longmapsto\rm{616= \dfrac{22}{7} \times r^2 \times 4}\\[/tex]

[tex]\longmapsto\rm{616 \times 7 = 22 \times r^2 \times 4}\\[/tex]

[tex]\longmapsto\rm{4312 = 88 \times r^2 }\\[/tex]

[tex]\longmapsto\rm{\cancel{\dfrac{4312}{88}} = r^2 }\\[/tex]

[tex]\longmapsto\rm{49 = r^2 }\\[/tex]

[tex]\longmapsto\rm{\sqrt{49} = r }\\[/tex]

[tex]\longmapsto\rm{7 \; cm = r }\\[/tex]

Now,

[tex]\longmapsto\rm{Diameter = 2r }\\[/tex]

[tex]\longmapsto\rm{Diameter = 2(7 \; cm) }\\[/tex]

[tex]\longmapsto\bf{Diameter = 14 \; cm}\\[/tex]

The required answer is 14 cm.

Put these numbers in order from greatest to least.
8
-2-
25
2.45
-0.84

Answers

Answer:

25, 2.45, 8, -0.84, -2

Step-by-step explanation:

negative is a least number

positive is a greater number

Positive number-8, 25, 2.45

Negative number-(-2), -0.84

ordering number from greatest to least:

25, 2.45, 8, -0.84, -2

-2 is smallest then -0.84 because 2 is bigger then 0.84. It is opposite with the positive number.

The bigger the positive number the biggest it is. While the bigger the negative number the smallest it is.

Answer:

Step-by-step explanation:

The numbers are:

● 8

● -2

● 25

● 2.45

● -0.84

To make it easy classify the positive numbers apart and the negatives ones alone

● 2.45<8< 25

● -2 < -0.84

25 is the greatest and -2 is the least

● 25 > 8 > 2.45 > -0.84 > -2

BRAINLEST Find the sum of the first 6 terms of the infinite series: 1 - 2 + 4 - 8+...

Answers

Answer:

-21

Step-by-step explanation:

1-2+4-8+16-32

=-21

Answer:

The sum of the first 6 terms of the infinite series will be - 21.

Step-by-step explanation:

In this case, the infinite geometric series 1 - 2 + 4 - 8 + ... is represented by the following summation,

[tex]\sum _{{k=0}}^{{n}}(-2)^{k}[/tex]

Therefore if we continue this pattern, the first 6 terms will be 1 - 2 + 4 - 8 + 16 - 32. Adding these terms,

1 - 2 + 4 - 8 + 16 - 32

= - 1 + 4 - 8 + 16 - 32

= 3 - 8 + 16 - 32 = - 5 + 16 - 32

= 11 - 32 = Solution : - 21

What is numbers 1-30 added all together

Answers

Answer:

465

Step-by-step explanation:

The sum of consecutive numbers has a formula, and it's

[tex]\frac{n(n+1)}{2}[/tex].

Where n is the amount of numbers.

From 1-30, it's 30 numbers, so:

[tex]\frac{30(30+1)}{2} \\\\\frac{30(31)}{2} \\\\\frac{930)}{2} \\\\\\465[/tex]

Hope this helped!

The numbers 1-30 added together is 465

3-(-4) answer the question

Answers

Answer:

7

Step-by-step explanation:

[tex]3-(-4) \\-\times - = +\\3+4 \\=7[/tex]

Answer:

7

Step-by-step explanation:

because you when multiply -1 by -4 u get positive 4 then 3 + 4 equals 7

Will give brainliest answer

Answers

It is A) Tamara’s work is correct

When graphed you can see that the function is even

According to the local union president, the mean gross income of plumbers in the Salt Lake City area follows a normal distribution with a mean of $48,000 and a population standard deviation of $2,000. A recent investigative reporter for KYAK TV found, for a sample of 49 plumbers, the mean gross income was $47,600. At the 0.05 significance level, is it reasonable to conclude that the mean income is not equal to $47,600? Determine the p value. State the Null and Alternate hypothesis: State the test statistic: State the Decision Rule: Show the calculation: What is the interpretation of the sample data? Show the P value

Answers

Answer:

Step-by-step explanation:

Given that:

population mean [tex]\mu[/tex] = 47600

population standard deviation [tex]\sigma[/tex] = 2000

sample size n = 49

Sample mean [tex]\over\ x[/tex] = 48000

Level of significance = 0.05

The null and the alternative hypothesis can be computed as follows;

[tex]H_0 : \mu = 47600 \\ \\ H_1 : \mu \neq 47600[/tex]

Using the table of standard normal distribution, the value of z that corresponds to the two-tailed probability 0.05 is 1.96. Thus, we will reject the null hypothesis if the value of the test statistics is less than -1.96 or more than 1.96.

The test statistics can be calculated by using the formula:

[tex]z= \dfrac{\overline X - \mu }{\dfrac{\sigma}{ \sqrt{n}}}[/tex]

[tex]z= \dfrac{ 48000-47600 }{\dfrac{2000}{ \sqrt{49}}}[/tex]

[tex]z= \dfrac{400 }{\dfrac{2000}{ 7}}[/tex]

[tex]z= 1.4[/tex]

Conclusion:

Since 1.4 is lesser  than 1.96 , we fail to reject the null hypothesis and  that there is insufficient information to conclude that the   mean gross income is not equal to $47600

The P-value is being calculate as follows:

P -value = 2P(Z>1.4)

P -value =  2 (1 - P(Z< 1.4)

P-value = 2 ( 1 - 0.91924)

P -value = 2 (0.08076 )

P -value = 0.16152

The temperature dropped 15 degrees in an hour. If the starting temperature was 10 degrees, What was the final temperature?​

Answers

Answer:

Step-by-step explanation:

15-10=5 degrees

44,587 plus what equals 65,000​

Answers

Let the missing number be x.

=> x + 44587 = 65000

=> x = 65000 - 44587

= 20413

Ans.= The missing number will be 20413.

Hope this answer helps you..

..

..

Select it as the BRAINLIEST..

Other Questions
What was designed to keep the Colonists from moving into the Ohio river valley? A small object has a mass of 68.1 grams. When completely immersed in a graduated cylinder with a water level of 25.0mL the object causes the water level to rise to 43.3mL. What is the density of the object in g/mL? Can all opportunity costs be evaluated using a cost/benefit analysis? Use an example to explain your answer surface area of a prism please help its my last day 120 points What type of relationship exists between the length of a wire and the resistance, if all other factors remain the same?OA.Resistance is directly related to length.OB.Resistance is directly related to the square of the length.O c. Resistance is inversely related to the length.OD.Resistance is inversely related to the square of the length. I WILL BE MARKING THE CORRECT ANSWER BRAINLEST PLS HELP When the distributive property is used to solve the equation 3 ( x + 5 ) = 5x - 7 what could the next step be? Use the PRESENT TENSE in the following sentences 1.- Yo ____________ (querer) que mis hijos estudien mucho2.- Nosotros ______________________ (cerrar) la ventana.3.- Antonio ______________________ (venir) a vernos.4.- Los doctores _____________________(hacer) ejercicios.5.- Mi sobrino __________________________ (ser) abogado.6.- Pedro__________________________ (amar) a Marta.7.-Ustedes no ____________________(tomar) vino.8.- Linda _____________(traer) el ordenador/la computadora.9.- Ella no ____________________(lavar) la ropa sucia.10.- Mis hijos ____________________ (poner) la mesa A doctor asks a nurse to give apatient 250 mg (milligrams) of thedrug Simethicone. The drug isavailable only in a solution whoseconcentration is 40 mg Simethiconeper 0.6 ml (milliliter) of solution.How many milliliters of solutionshould the nurse give the patient? find the measure of a Last school year, in the school of Business Administration, 30% were Accounting majors, 24% Management majors, 26% Marketing majors, and 20% Economics majors. A sample of 300 students taken from this year's students of the school showed the following number of students in each major: Accounting 83 Management 68 Marketing 85 Economics 64 Total 300 Has there been any significant change in the number of students in each major between the last school year and this school year? According to our textbook, the original audience for the Gospel of John seems to be Greek-speaking Jews who were living outside of Israel.a. trueb. false What is the most precise name for quadrilateral ABCD with vertices A(5,2), B(3, 5),C(4, 5),and D(2, 2)? A round wading pool has a diameter of 115cm deep. A hose is filling the pool at a rate of 34000cm^3, cubed per minute. How long will it take to fill the pool to a depth of 20cm? Calculate the perimeter of a semi-circlewhose diameter is 14cm What statement is accurate based on the study of tree rings?O Trees near the arctic will have thicker rings than those near the equator.Trees with a pattern of thin rings indicate a wet, warm climate.Size and density of tree rings can give information on past climates.O The number of rings indicate how much fruit the tree can bear. A firm has current assets of $36,000, cash of $5,000, current liabilities of $20,000, total assets of $80,000 and total liabilities of $45,000. What is its net working capital? a. $16,000 b. $28,000 c. $35,000 d. $44,000 "Rihanna Company is considering purchasing new equipment for $379,200. It is expected that the equipment will produce net annual cash flows of $48,000 over its 10-year useful life. Annual depreciation will be $37,920. Compute the cash payback period. (Round answer to 1 decimal place, e.g. 10.5.)" Simplify:[tex] \frac{ {a}^{5} {b}^{8} }{ {a}^{4} {b}^{10} } [/tex]HELP! answer if you can ! Help please!!! Thxxxxx The tomb of Rudolf of Swabia, shown below, is an example of a